Sei sulla pagina 1di 26

Batangas State University

Mech 303 Mechanics of Deformable Bodies

Submitted by:

Talabis, Jan Bredyl E.

EE-3202

Submitted to:

Engr. Jonas S. De Castro

Instructor

February 10, 2017


Batangas State University

Mech 303 Mechanics of Deformable Bodies

Submitted by:

Justiniani, Karen Zhyne M.

EE-3202

Submitted to:

Engr. Jonas S. De Castro

Instructor

February 10, 2017

Normal And Shear Stress


1. A rectangular piece of Jello, h=2 inches tall by
L=6 inches wide, sits on a plate with a piece of
aluminum foil on top. The foil is moved to the right by
w=0.1 inch, with the Jello sticking to it.

Req'd: What is the shear strain in the Jello?

Sol'n: The shear strain is the reduction in right angle (in


radians):

g = w/h= (0.1 in)/(2 in) = g = 0.05

This value (5%) is very much larger than desired in


engineering situations.

2. A pipe of diameter D=1.0 inches and thickness


t=0.050 inches is twisted by a wrench with a torque
of T = Pd = 20 in-lb.

Req'd: Estimate the shear stress in the thin-walled pipe due


ONLY the Torque. Hint: Area = pDt = 2pRt = circumference
times thickness.

Sol'n: Shear stress for the thin-walled pipe is:

t = (T/R)/A = T/[R2pRt] = [2T]/[pD2t]


= [(2)(20 in-lb)] / [p(1 in)2(0.05 in)]

t = 255 psi = 260 psi (2 significant digits)

3. A 200-mm-diameter pulley is prevented from rotating relative to 60-mm-diameter


shaft by a 70-mm-long key, as shown in Fig. P-118. If a torque T = 2.2 kNm is
applied to the shaft, determine the width b if the allowable shearing stress in the
key is 60 MPa.

T=0.03FT=0.03F

2.2=0.03F2.2=0.03F
F=73.33kNF=73.33kN

V=AV=A
Where:
V=F=73.33kNV=F=73.33kN
A=70bA=70b
=60MPa=60MPa
Thus,
73.33(1000)=60(70b)73.33(1000)=60(70b)
b=17.46mmb=17.46mm

4. A block of gelatin is 60 mm by 60 mm by 20 mm when


unstressed. A force of .245 N is applied tangentially to the upper surface causing
a 5 mm displacement relative to the lower surface. The block is placed such that
60X60 comes on the lower and upper surface. Find the shearing stress and
shearing strain.

(a) (68.1 N/m2 , .25 , 272.4 N/m2)


(b) (68 N/m2 , .25 , 272 N/m2)
(c) (67 N/m2 , .26 , 270.4 N/m2)
(d) (68.5 N/m2 , .27 , 272.4 N/m2)

Solution:

Shear stress=FA=.24536104=68.1N/m2=FA=.24536104 = 68.1N/m2

Shear strain=tan=dh=520=.25=tan=dh =520 =.25

5. An aluminum rod is rigidly attached between a steel rod and a bronze rod as shown
in Fig. P-108. Axial loads are applied at the positions indicated. Find the maximum value
of P that will not exceed a stress in steel of 140 MPa, in aluminum of 90 MPa, or in
bronze of 100 MPa.

For bronze:
brAbr=2PbrAbr=2P
100(200)=2P100(200)=2P
P=10000NP=10000N
For aluminum:
alAal=PalAal=P
90(400)=P90(400)=P
P=36000NP=36000N

For Steel:
stAst=5PstAst=5P
P=14000NP=14000N

For safe value of P, use the smallest above. Thus,


P=10 000 N=10 kNP=10 000 N=10 kN

6. Determine the largest weight W that can be supported by two wires shown in Fig. P-
109. The stress in either wire is not to exceed 30 ksi. The cross-sectional areas of wires
AB and AC are 0.4 in2 and 0.5 in2, respectively.

Free body diagram of Joint A

For wire AB: By sine law (from the force polygon):


TABsin40=Wsin80TABsin40=Wsin80
TAB=0.6527WTAB=0.6527W
ABAAB=0.6527WABAAB=0.6527W
30(0.4)=0.6527W30(0.4)=0.6527W
W=18.4kipsW=18.4kips

For wire AC:


TACsin60=Wsin80TACsin60=Wsin80
TAC=0.8794WTAC=0.8794W
TAC=ACAACTAC=ACAAC
0.8794W=30(0.5)0.8794W=30(0.5)
W=17.1kipsW=17.1kips

For safe load W,


W=17.1 kipsW=17.1 kips

7. A 12-inches square steel bearing plate lies between an 8-inches diameter wooden
post and a concrete footing as shown in Fig. P-110. Determine the maximum value of
the load P if the stress in wood is limited to 1800 psi and that in concrete to 650 psi.

For wood:
Pw=wAwPw=wAw
Pw=1800[14(82)]Pw=1800[14(82)]
Pw=90477.9lbPw=90477.9lb

From FBD of Wood:

P=Pw=90477.9lbP=Pw=90477.9lb

For concrete:
Pc=cAcPc=cAc
Pc=650(122)Pc=650(122)
Pc=93600lbPc=93600lb
From FBD of Concrete:

P=Pc=93600lbP=Pc=93600lb

For safe load P,


P=90478lbP=90478lb

8. For the truss shown in Fig. P-111, calculate the stresses in members CE, DE, and DF.
The cross-sectional area of each member is 1.8 in2. Indicate tension (T) or compression
(C).

From the FBD of the truss:

MA=0MA=0
24RF=16(30)24RF=16(30)
RF=20kRF=20k

At joint F:

FV=0FV=0
35DF=2035DF=20
DF=3313k(Compression)DF=3313k(Compression)

At joint D:

By symmetry
BD=DF=3313k(Compression)BD=DF=3313k(Compression)

FV=0FV=0
DE=35BD+35DFDE=35BD+35DF
DE=35(3313)+35(3313)DE=35(3313)+35(3313)
DE=40k(Tension)DE=40k(Tension)

At joint E:

FV=0FV=0
35CE+30=4035CE+30=40
CE=1623k(Tension)CE=1623k(Tension)
Stresses: (Stress = Force/Area)
CE=16231.8=9.26ksi (Tension)CE=16231.8=9.26ksi (Tension)
DE=401.8=22.22ksi (Tension)DE=401.8=22.22ksi (Tension) answer
DF=33131.8=18.52ksi (Compression)DF=33131.8=18.52ksi (Compression)

9. Find the stresses in members BC, BD, and CF for the truss shown in Fig. P-113.
Indicate the tension or compression. The cross sectional area of each member is 1600
mm2.

For member BD: (See FBD 01)


MC=0MC=0
3(45BD)=3(60)3(45BD)=3(60)
BD=75kNBD=75kN Tension

BD=BDABD=BDA
75(1000)=BD(1600)75(1000)=BD(1600)
BD=46.875MPa (Tension)BD=46.875MPa (Tension) answer

For member CF: (See FBD 01)


MD=0MD=0
4(12CF)=4(90)+7(60)4(12CF)=4(90)+7(60)
CF=275.77kNCF=275.77kN Compression

CF=CFACF=CFA
275.77(1000)=CF(1600)275.77(1000)=CF(1600)
CF=172.357MPa (Compression)CF=172.357MPa (Compression) answer

For member BC: (See FBD 02)

MD=0MD=0
4BC=7(60)4BC=7(60)
BC=105kNBC=105kN Compression

BC=BCABC=BCA
105(1000)=BC(1600)105(1000)=BC(1600)
BC=65.625MPa (Compression)BC=65.625MPa (Compression) answer

10. A homogeneous 800 kg bar AB is supported at either end by a cable as shown in


Fig. P-105. Calculate the smallest area of each cable if the stress is not to exceed 90
MPa in bronze and 120 MPa in steel.

Solution 105
HideClick here to show or hide the solution
By symmetry:
Pbr=Pst=12(7848)Pbr=Pst=12(7848)
Pbr=3924 NPbr=3924 N
Pst=3924 NPst=3924 N
For bronze cable:
Pbr=brAbrPbr=brAbr
3924=90Abr3924=90Abr
Abr=43.6mm2Abr=43.6mm2 answer

For steel cable:


Pst=stAstPst=stAst
3924=120Ast3924=120Ast
Ast=32.7mm2Ast=32.7mm2

A homogeneous 800 kg bar AB is supported at either end by a cable as shown in Fig. P-


105. Calculate the smallest area of each cable if the stress is not to exceed 90 MPa in
bronze and 120 MPa in steel.

By symmetry:
Pbr=Pst=12(7848)Pbr=Pst=12(7848)
Pbr=3924 NPbr=3924 N
Pst=3924 NPst=3924 N

For bronze cable:


Pbr=brAbrPbr=brAbr
3924=90Abr3924=90Abr
Abr=43.6mm2Abr=43.6mm2 answer
For steel cable:
Pst=stAstPst=stAst
Stress on thin walled pressure vessels

Problem-1:

A cylinder is 300 mm mean diameter with a wall 2 mm thick. Calculate the maximum

pressure difference allowed between the inside and outside if the stress in the wall must

not exceed 150 MPa.

Solution:

The solution must be based on the circumferential stress since this is the largest.

c = pD/2t = 150 MPa

p = 150 MPa x 2t/D = 150 x 2 x 0.002/0.3

p = 2 MPa

Problem-2:

. A cylindrical pressure vessel is fabricated from steel plating that has a thickness of 20
mm. The diameter of the pressure vessel is 450 mm and its length is 2.0 m. Determine
the maximum internal pressure that can be applied if the longitudinal stress is limited to
140 MPa, and the circumferential stress is limited to 60 MPa.

Based on circumferential stress (tangential):

FV=0FV=0

F=2TF=2T

p(DL)=2(tLt)p(DL)=2(tLt)

t=pD2tt=pD2t

60=p(450)2(20)60=p(450)2(20)

p=5.33MPap=5.33MPa

Based on longitudinal stress:


FH=0FH=0

F=PF=P

p(14D2)=l(D)p(14D2)=l(D)

l=pD4tl=pD4t

140=p(450)4(20)140=p(450)4(20)

p=24.89MPap=24.89MPa

Use p=5.33MPap=5.33MPa answer

Problem-3:

Calculate the minimum wall thickness for a cylindrical vessel that is to carry a gas at a
pressure of 1400 psi. The diameter of the vessel is 2 ft, and the stress is limited to 12
ksi.

The critical stress is the tangential stress


t=pD2tt=pD2t

12000=1400(212)2t12000=1400(212)2t

t=1.4 int=1.4 in answer

Problem-4:

The wall thickness of a 4-ft-diameter spherical tank is 5/16 inch. Calculate the allowable
internal pressure if the stress is limited to 8000 psi.
Total internal pressure:
P=p(14D2)P=p(14D2)

Resisting wall:
F=PF=P

A=p(14D2)A=p(14D2)

(Dt)=p(14D2)(Dt)=p(14D2)

=pD4t=pD4t

8000=p(412)4(516)8000=p(412)4(516)

p=208.33psip=208.33psi answer

A water tank, 22 ft in diameter, is made from steel plates that are 1/2 in. thick. Find the
maximum height to which the tank may be filled if the circumferential stress is limited to
6000 psi. The specific weight of water is 62.4 lb/ft 3.

t=6000psi=6000lb/in2(12in/ft)2t=6000psi=6000lb/in2(12in/ft)2

t=864000lb/ft2t=864000lb/ft2
Assuming pressure distribution to be uniform:
p=h=62.4hp=h=62.4h

F=pA=62.4h(Dh)F=pA=62.4h(Dh)

F=62.4(22)h2F=62.4(22)h2

F=1372.8h2F=1372.8h2

T=tAt=864000(th)T=tAt=864000(th)

T=864000(12112)hT=864000(12112)h

T=36000hT=36000h

F=0F=0

F=2TF=2T

1372.8h2=2(36000h)1372.8h2=2(36000h)
h=52.45fth=52.45ft answer

Based on actual pressure distribution:

Total hydrostatic force, F:


FF = volume of pressure diagram

F=12(h2)D=12(62.4h2)(22)F=12(h2)D=12(62.4h2)(22)

F=686.4h2F=686.4h2

MA=0MA=0
2T(12h)F(13h)=02T(12h)F(13h)=0

T=13FT=13F

t(ht)=13(686.4h2)t(ht)=13(686.4h2)

h=3tt686.4=3(864000)(12112)686.4h=3tt686.4=3(864000)
(12112)686.4

h=157.34ft

5. A pipe carrying steam at 3.5 MPa has an outside diameter of 450 mm and a wall
thickness of 10 mm. A gasket is inserted between the flange at one end of the
pipe and a flat plate used to cap the end. How many 40-mm-diameter bolts must
be used to hold the cap on if the allowable stress in the bolts is 80 MPa, of which
55 MPa is the initial stress? What circumferential stress is developed in the pipe?
Why is it necessary to tighten the bolt initially, and what will happen if the steam
pressure should cause the stress in the bolts to be twice the value of the initial
stress?

F=AF=A

F=3.5[14(4302)]F=3.5[14(4302)]

F=508270.42NF=508270.42N

P=FP=F

(boltA)n=508270.42N(boltA)n=508270.42N

(8055)[14(402)]n=508270.42(8055)[14(402)]n=508270.42

n=16.19n=16.19 say 17 bolts answer

Circumferential stress (consider 1-m strip):


F=pA=3.5[430(1000)]F=pA=3.5[430(1000)]
F=1505000NF=1505000N

2T=F2T=F

2[t(1000)(10)]=15050002[t(1000)(10)]=1505000

t=75.25MPat=75.25MPa answer

The tank shown in Fig. P-141 is fabricated from 1/8-in steel plate. Calculate the
maximum longitudinal and circumferential stress caused by an internal pressure of 125
psi.

Longitudinal Stress:

F=pA=125[1.5(2)+14(1.5)2](122)F=pA=125[1.5(2)+14(1.5)2](122)

F=85808.62lbsF=85808.62lbs

P=FP=F
l[2(212)(18)+(1.512)(18)]=85808.62l[2(212)(18)+(1.512)(18)]=85808.62

l=6566.02psil=6566.02psi

l=6.57ksil=6.57ksi answer

Circumferential Stress:

F=pA=125[(212)L+2(0.7512)L]F=pA=125[(212)L+2(0.7512)L]

F=5250L lbsF=5250L lbs

2T=F2T=F

2[t(18)L]=5250L2[t(18)L]=5250L

t=21000psit=21000psi

t=21ksit=21ksi

At what angular velocity will the stress of the rotating steel ring equal 150 MPa if its
mean radius is 220 mm? The density of steel 7.85 Mg/m 3.

CF=M2xCF=M2x

Where:
M=V=ARM=V=AR

x=2R/x=2R/

Thus,
CF=AR2(2R/)CF=AR2(2R/)

CF=2AR22CF=2AR22
2T=CF2T=CF

2A=2AR222A=2AR22

=R22=R22

From the given (Note: 1 N = 1 kgm/sec2):


=150MPa=150MPa

=150000000kgm/sec2m2=150000000kgm/sec2m2

=150000000kg/msec2=150000000kg/msec2

=7.85Mg/m3=7850kg/m3=7.85Mg/m3=7850kg/m3

R=220mm=0.22mR=220mm=0.22m

Therefore,
150000000=7850(0.22)22150000000=7850(0.22)22

=628.33rad/sec=628.33rad/sec answe

Find the limiting peripheral velocity of a rotating steel ring if the allowable stress is 20 ksi
and steel weighs 490 lb/ft3. At what revolutions per minute (rpm) will the stress reach 30
ksi if the mean radius is 10 in.?

Centrifugal Force, CF:


CF=M2xCF=M2x

where:
M=Wg Vg=RAqM=Wg Vg=RAq

=v/R=v/R

x=2R/x=2R/

Thus,
CF=RAq(vR)2(2R)CF=RAq(vR)2(2R)

CF=2Av2gCF=2Av2g

2T=CF2T=CF

2A=2Av2g2A=2Av2g

=v2g=v2g
From the given data:
=20ksi=(20000lb/in2)(12in/ft)a2=20ksi=(20000lb/in2)(12in/ft)a2

=2880000lb/ft2=2880000lb/ft2

=490lb/ft3=490lb/ft3

2880000=490v232.22880000=490v232.2

v=435.04ft/secv=435.04ft/sec answer

When =30ksi=30ksi, and R=10inR=10in


=v2g=v2g

30000(122)=490v232.230000(122)=490v232.2

v=532.81ft/secv=532.81ft/sec

=v/R=532.8110/12=v/R=532.8110/12

=639.37rad/sec=639.37rad/sec

=639.37radsec1rev2rad60sec1min=639.37radsec1rev2rad60sec1min

=6,105.54rpm=6,105.54rpm answer

Problem 138
The strength of longitudinal joint in Fig. 1-17 is 33 kips/ft, whereas for the girth is 16
kips/ft. Calculate the maximum diameter of the cylinder tank if the internal pressure is
150 psi.

For longitudinal joint (tangential stress):


Consider 1 ft length
F=2TF=2T

pD=2ttpD=2tt

t=pD2tt=pD2t

33000t=21600D2t33000t=21600D2t

D=3.06ft=36.67in.D=3.06ft=36.67in.

For girth joint (longitudinal stress):

F=PF=P

p(14D2)=l(Dt)p(14D2)=l(Dt)

l=pD4tl=pD4t

16000t=21600D4t16000t=21600D4t

D=2.96ft=35.56in.D=2.96ft=35.56in.

Use the smaller diameter, D=35.56in.D=35.56in.


Shearing deformation

E 29 10 6 psi
D 1.07 in. d 0.618 in.

Determine the deformation of the steel rod shown under the given loads.

SOLUTION:

Divide the rod into three components:

L3 16 in. L1 L2 12 in.
A3 0.3 in A1 A2 0.9 in 2
2

Apply free-body analysis to each component to determine internal forces,

P1 60 103 lb

P2 15 103 lb

P3 30 103 lb
Pi Li 1 P1L1 P2 L2 P3 L3

A
i i iE E A
1 A 2 A 3


1
60 103 12 15 103 12 30 103 16


29 106 0.9 0.9 0.3

75.9 10 3 in.

75.9 10 3 in.
TherigidbarBDEissupportedbytwolinksABandCD.LinkABismadeof
aluminum(E=70GPa)andhasacross-sectionalareaof500mm2.LinkCDis
madeofsteel(E=200GPa)andhasacross-sectionalareaof(600mm2).Forthe
30-kNforceshown,determinethedeflectiona)ofB,b)ofD,andc)ofE.

Solution:

Free body: Bar BDE

MB 0
0 30 kN 0.6 m FCD 0.2 m
FCD 90 kN tension
MD 0
0 30 kN 0.4 m FAB 0.2 m
FAB 60 kN compression
Displacement of D:
:
PL
D
AE
90 103 N 0.4 m

600 10-6 m2 200 109 Pa
300 10 6 m

D 0.300 mm

DisplacementofB:

PL
B
AE
60 103 N 0.3 m

500 10-6 m2 70 109 Pa
514 10 6 m
B 0.514 mm
9".extensionofsection(1)=63.66x106x210x109=75.8x10-6mextensionof
section(2)=22.2x106x100x10-3210x109=10.6x10-6mextensionof
section(3)=113.2x106400x10-3210x109=215.6x10-6mtotalextension
=(75.8+10.6+215.6)10-6=302xlO-6m=0.302mm

Potrebbero piacerti anche